Which expression has the same value as the one below?
38 + (-18)
O A. 38
O B. 38 - 18
O C. 38 + 18
O D. 56

Answers

Answer 1

Answer:

answer is B 38-18

Step-by-step explanation:

38 + (-18)

38-18

Answer 2
I believe the answer is B

38+ (-18) turns into 38-18 because after removing the parentheses around -18 there is no need for the +

Related Questions

What is the probability that a randomly selected day in the summer will be rainy if it’s cloudy?

Answers

Answer:

0.872

Step-by-step explanation:

Given that :

P(cloudy) = P(C) = 0.94

P(cloudy and rainy) = P(C n R) = 0.82

Probability that a given day will be rainy if it is cloudy ; this is a conditional probability problem:

Recall ; P(A|B) = P(AnB) / P(B)

P(R|C) = P(C n R) / P(C) = 0.82 / 0.94 = 0.872

Anya contributed $1,200 toward the purchase
of a $2,000 computer. Her brother contributed
$240 toward the same computer. Her parents
provided the rest of the money for the computer. What percentage of the total cost of the computer did Anya's parents pay?

Answers

Answer:

Parent's share would be : 28%

Step-by-step explanation:

Cost of the computer = $2000

Anya's share = $1200

Brother's share = $240

Parents paid rest. That will be : 2000 - 1200 - 240 = $560

Percentage of parents share :

                                             [tex]\frac{560}{2000} \times 100 = 28 \%[/tex]

Use logarithmic differentiation to differentiate the question below
[tex]y = x \sqrt[3]{1 + {x}^{2} } [/tex]

Answers

Answer:

[tex] \orange{ \bold{\frac{dy}{dx} =\frac{ 5{x}^{2} + 3 }{3\sqrt[3]{(1 + {x}^{2})^{2} } }}}[/tex]

Step-by-step explanation:

[tex]y = x \sqrt[3]{1 + {x}^{2} } \\ assuming \: log \: both \: sides \\log y = log(x \sqrt[3]{1 + {x}^{2} } ) \\ \therefore log y = logx + log(\sqrt[3]{1 + {x}^{2} } ) \\ \therefore log y = logx + \frac{1}{3} log({1 + {x}^{2} } ) \\ differentiating \: both \: sides \: w.r.t.x \\ \frac{1}{y} \frac{dy}{dx} = \frac{1}{x} + \frac{1}{3} . \frac{1}{(1 + {x}^{2}) } (0 + 2x) \\ \frac{1}{y} \frac{dy}{dx} = \frac{1}{x} + \frac{2x}{3(1 + {x}^{2}) }\\ \frac{1}{y} \frac{dy}{dx} =\frac{3(1 + {x}^{2}) + 2 {x}^{2} }{3x(1 + {x}^{2}) }\\ \frac{1}{y} \frac{dy}{dx} =\frac{3 + 3{x}^{2} + 2 {x}^{2} }{3x(1 + {x}^{2}) }\\ \frac{1}{y} \frac{dy}{dx} =\frac{3 + 5{x}^{2} }{3x(1 + {x}^{2}) }\\ \frac{dy}{dx} =\frac{y(3 + 5{x}^{2} )}{3x(1 + {x}^{2}) } \\ \\ \frac{dy}{dx} =\frac{x \sqrt[3]{1 + {x}^{2} } (3 + 5{x}^{2} )}{3x(1 + {x}^{2}) }\\ \\ \frac{dy}{dx} =\frac{(3 + 5{x}^{2} )\sqrt[3]{1 + {x}^{2} } }{3(1 + {x}^{2}) }\\ \\ \purple{ \bold{\frac{dy}{dx} =\frac{ 5{x}^{2} + 3 }{3\sqrt[3]{(1 + {x}^{2})^{2} } }}}[/tex]

5x + 3y = 17
-8x - 3y = 9

Answers

Answer:

1)x=−3/5y+17/5  

2)x=−3/8y+−9/8

Step-by-step explanation:

Let's solve for x.

5x+3y=17

Step 1: Add -3y to both sides.

5x+3y+−3y=17+−3y

5x=−3y+17

Step 2: Divide both sides by 5.

5x/5=−3y+17/5

x=−3/5y+17/5

Answer:

x=−3/5y+17/5  

Let's solve for x.

−8x−3y=9

Step 1: Add 3y to both sides.

−8x−3y+3y=9+3y

−8x=3y+9

Step 2: Divide both sides by -8.

−8x/−8=3y+9/−8

x=−3/8y+−9/8

Answer:

x=−3/8y+−9/8

I HOPE THIS IS CORRECT IF NOT TELL ME AND ILL FIX IT.

Michael and Sondra are mixing lemonade. In Michael’s lemonade, the ratio of lemons to water is 1:4. In Sondra’s lemonade, the ratio of lemons to water is 2:6. Several equivalent ratios for each mixture are shown in the ratio tables.

Michael
Lemons
Cups of Water
1 4
3 12
4 16
Sondra
Lemons
Cups of Water
2 6
4 12
6 18

Imagine that you want to compare Michael’s ratio to Sondra’s ratio. Which two ratios in the tables shown have a common denominator you could use to compare?

Answers

Answer:

1/4= 3/12 & 2/6 = 4/12

Step-by-step explanation:

Answer:

Its simply B

Step-by-step explanation:

If we are to express both ratios in their simplest form, we will have the ratio of Michael’s lemonade is 1:4 and that of Sondra is 1:3. The denominator that can be used in order to compare the ratios is that which can be divided by both ratios. For example, we have 12 as a denominator. The ratios can be expressed as 3/12 and 4/12. Also, the denominator can be 24 such that the ratios can be expressed as 6/24 and 8/24.

What is the solution to the following system of equations?
[3x-2y = 12
16x-4y = 24
O It has infinitely many solutions.
It has no solution.
It has one solution (2, -3).
It has one solution (4,0).

Answers

Answer:

(0, -6)

Step-by-step explanation:

Given the following systems of linear equations;

3x - 2y = 12 ...... equation 1

16x - 4y = 24 ........ equation 2

We would solve for the solution using the elimination method;

Multiplying eqn 1 by 2, we have;

2 * (3x - 2y = 12)

6x - 4y = 24

16x - 4y = 24

Subtracting the two equations, we have;

(6x - 16x) + (-4y -[-4y]) = (24 - 24)

-10x - 0 = 0

-10x = 0

x = -0/10 = 0

Next, we would find the value of y;

3x - 2y = 12

3(0) - 2y = 12

0 - 2y = 12

-2y = 12

y = -12/2

y = -6

Check:

3x - 2y = 12

3(0) - 2(-6) = 12

0 - (-12) = 12

12 = 12

Note: the options provided for this questions are incorrect or inappropriate.

Only answer if you're very good at Math.

What is the sum of 7x/x^2 - 4 and 2/x + 2?

A: 7x + 2/x^2 - 4

B: 9x - 4/x^2 - 4

C: 7x + 2/ x^2 + x - 2

D: 9/x

Answers

Answer:

Solution given:

B: 9x - 4/x^2 - 4

Step-by-step explanation:

.k

Geometry, circles please help that'd be great.

Answers

Answer:

maybe the answer is 63cm...

Can y’all help me on question 2?!

Answers

Answer:

4.5

Step-by-step explanation:

18 divided by 4 equals 4.5

gold that is 24 karat is 100% pure gold. gold that is 14 karat is 14 parts pure gold and 10 parts another metal, such as copper,zinc,silver, or nickel. What percent of 14 karat gold if pure gold.

help im confused

Answers

It would be 58.3%

100% is 14+10=24

14 divided by 24 = 0.583
Move decimal over to the right twice is 58.3%

what is the slope of the line that passes through these two points?​

Answers

Answer:

slope of the line is 0

Step-by-step explanation:

given points are:

(-3 , 2)=(x1 , y1)

(4 , 2)=(x2 , y2)

slope =y2 - y1/x2 - x1

=2-2/4-(-3)

=0/4+3

=0/7

=0

please answer both questions thanks ​

Answers

Answer:

8. cos(58) = 20/x

x · cos(58) = 20

x = 20/cos(58)

= 37.742

≈ 37.7

9. tan(69) = 324/x

x · tan(69) = 324

x = 324/tan(69)

= 124.372

≈ 124.4

You will purchase some cds and dvds. If you purchase 13 cds and 5 dvds, it will cost you $96.70; if you purchase 5 cds and 12 dvds, it will cost you $134.50. Write and solve a system of equations to solve.

Answers

Question

You will purchase some cds and dvds. If you purchase 13 cds and 5 dvds, it will cost you $96.70; if you purchase 5 cds and 12 dvds, it will cost you $134.50. Write and solve a system of equations to solve for the cost of each cd and the cost of each dvd.

Answer:

Cost of cd = $3.72

Cost of dvd = $9.66

Step-by-step explanation:

Let the cost of a cd be c

Let the cost of a dvd be d

From the second statement:

If you purchase 13 cds and 5 dvds, it will cost you $96.70

This means that

13c + 5d = 96.70            ----------------(i)

Also, from the third statement:

if you purchase 5 cds and 12 dvds, it will cost you $134.50

This means that;

5c + 12d = 134.50            ---------------------(ii)

The equations to solve are equations (i) and (ii)

13c + 5d = 96.70

5c + 12d = 134.50

Multiply the first equation by 5 and the second equation by 13. i.e

5 x (13c + 5d = 96.70)

13 x (5c + 12d = 134.50)

This will give

65c + 25d = 483.5

65c + 156d = 1748.5

Find the difference of the two equations

    65c + 25d = 483.5

-

    65c + 156d = 1748.5

    0   -  131d   = - 1265

This gives;

-131d = -1265

Divide both sides by -131

[tex]\frac{-131d}{-131} = \frac{-1265}{-131}[/tex]

d = 9.66

This means that the cost of a dvd which is d = $9.66

Now substitute the value of d = 9.66 into equation (i) as follows;

13c + 5(9.66) = 96.70  

Expand the above and solve for c

13c + 48.3 = 96.70

13c = 48.4

c = [tex]\frac{48.4}{13}[/tex]

c = 3.72

This means that the cost of a cd which is c = $3.72

There are two machines available for cutting corks intended for use in wine bottles. The first produces corks with diameters that are normally distributed with mean 3 cm and standard deviation 0.08 cm. The second machine produces corks with diameters that have a normal distribution with mean 3.04 cm and standard deviation 0.03 cm. Acceptable corks have diameters between 2.9 cm and 3.1 cm.
What is the probability that the first machine produces an acceptable cork? (Round your answer to four decimal places.)
What is the probability that the second machine produces an acceptable cork? (Round your answer to four decimal places.)
Please explain the math behind your answer so I am able to understand!(:

Answers

Answer:

0.7888 = 78.88% probability that the first machine produces an acceptable cork.

0.9772 = 97.72% probability that the second machine produces an acceptable cork.

Step-by-step explanation:

Normal Probability Distribution:

Problems of normal distributions can be solved using the z-score formula.

In a set with mean [tex]\mu[/tex] and standard deviation [tex]\sigma[/tex], the z-score of a measure X is given by:

[tex]Z = \frac{X - \mu}{\sigma}[/tex]

The Z-score measures how many standard deviations the measure is from the mean. After finding the Z-score, we look at the z-score table and find the p-value associated with this z-score. This p-value is the probability that the value of the measure is smaller than X, that is, the percentile of X. Subtracting 1 by the p-value, we get the probability that the value of the measure is greater than X.

First machine:

Mean 3 cm and standard deviation 0.08 cm, which means that [tex]\mu = 3, \sigma = 0.08[/tex]

What is the probability that the first machine produces an acceptable cork?

This is the p-value of Z when X = 3.1 subtracted by the p-value of Z when X = 2.9. So

X = 3.1

[tex]Z = \frac{X - \mu}{\sigma}[/tex]

[tex]Z = \frac{3.1 - 3}{0.08}[/tex]

[tex]Z = 1.25[/tex]

[tex]Z = 1.25[/tex] has a p-value of 0.8944

X = 2.9

[tex]Z = \frac{X - \mu}{\sigma}[/tex]

[tex]Z = \frac{2.9 - 3}{0.08}[/tex]

[tex]Z = -1.25[/tex]

[tex]Z = -1.25[/tex] has a p-value of 0.1056

0.8944 - 0.1056 = 0.7888

0.7888 = 78.88% probability that the first machine produces an acceptable cork.

What is the probability that the second machine produces an acceptable cork?

For the second machine, [tex]\mu = 3.04, \sigma = 0.03[/tex]. Now to find the probability, same procedure.

X = 3.1

[tex]Z = \frac{X - \mu}{\sigma}[/tex]

[tex]Z = \frac{3.1 - 3.04}{0.03}[/tex]

[tex]Z = 2[/tex]

[tex]Z = 2[/tex] has a p-value of 0.9772

X = 2.9

[tex]Z = \frac{X - \mu}{\sigma}[/tex]

[tex]Z = \frac{2.9 - 3.04}{0.03}[/tex]

[tex]Z = -4.67[/tex]

[tex]Z = -4.67[/tex] has a p-value of 0

0.9772 - 0 = 0.9772

0.9772 = 97.72% probability that the second machine produces an acceptable cork.

Sand is being dumped from a conveyor belt and forms a conical pile. Assuming that the height of this cone is always exactly 3 times the size of the radius of its base, and that thesand is added at the rate of 10 m^3/min, how fast is the height increasing when the pile is15 m high?

Answers

Answer:

dh/dt =  0.4 m/min

Step-by-step explanation:

The volume of the cone is:

V(c) = (1/3)*r² *h              if always  h = 3r    then    r = h/3

The volume of the cone as a function of h will be:

V(h)  =  (1/3)* (h/3)²*h

V(h) =  (1/27)*h³

The increasing rate of the volume is equal to the rate of sand added the:

D(V)/dt   = (1/27)*3*h²*dh/dt

D(v) / dt  =  10 m³/min

h =  15 m      and   dh/dt is the rate of increasing of the height

By substitution

10 m³/min  = ( 1/9)* 225 * dh/dt  (m²)

dh/dt =  90 / 225   m/min

dh/dt =  0.4 m/min

What is the value........

Answers

Answer:

[tex]a_5 = 120[/tex]

Step-by-step explanation:

Given

[tex]a_1 = 1[/tex]

[tex]a_n = n(a_{n-1})[/tex]

Required

[tex]a_5[/tex]

This is calculated as:

[tex]a_5 = 5(a_{5-1})[/tex]

[tex]a_5 = 5*a_4[/tex]

Calculate [tex]a_4[/tex]

[tex]a_4 =4(a_{4-1})[/tex]

[tex]a_4 =4*a_3[/tex]

Calculate [tex]a_3[/tex]

[tex]a_3 =3*a_2[/tex]

Calculate [tex]a_2[/tex]

[tex]a_2 = 2 * a_1[/tex]

[tex]a_2 = 2 * 1[/tex]

[tex]a_2 = 2[/tex]

So:

[tex]a_3 =3*a_2[/tex]

[tex]a_3 = 3 * 2 = 6[/tex]

So:

[tex]a_4 =4*a_3[/tex]

[tex]a_4 = 4 * 6 =24[/tex]

Lastly;

[tex]a_5 = 5*a_4[/tex]

[tex]a_5 = 5 * 24[/tex]

[tex]a_5 = 120[/tex]

why are chiken nugget rw that come from plans

Answers

Answer:

because i decided that

Step-by-step explanation:

A store only sells 20-pound bags of ice. Over the weekend, the store sells 800 bags of ice, making $3,400. On Monday, the store sells 80 bags of ice. How much money does the store make selling ice on Monday?

Answers

£3400 / 800 bags of ice
= £4.25 per bag

80 X £4.25 = £340

The amount of money the store makes selling ice by the same ratio will be $340.

What are the ratio and proportion?

The ratio is the division of the two numbers.

For example, a/b, where a will be the numerator and b will be the denominator.

Proportion is the relation of a variable with another. It could be direct or inverse.

As per the given,

The store sells 800 bags of ice, making $3,400.

Ratio = 3400/800

With the same ratio,

The store sells x amount of bags of ice,

x/80 = 3400/800

x = $340

Hence "The store will make $340 from selling ice using the same ratio".

For more information about ratios and proportions,

brainly.com/question/26974513

#SPJ2

find the area of the following figure.

Answers

39 square meters
Explanation:
Divide figure into two triangles:

Area of triangle to the left (1/2•base•height):
1/2 • 6 • 4 = 12 square meters

Area of triangle to the right:
1/2 • 6 • 9 = 27 square meters

Add the area of both triangles:
27 + 12 = 39
FINAL ANSWER: 39 square meters
Hope this helps!

Use the method of least squares to solve the following problem.
Given the data set below, find the line of best fit? Then find the y-value for when x=7. Yes, there are supposed to be
two 6's.
Х
1
2
4
5
6
6
8
9
Y
14
10
12
8
9
6
3
4

Answers

purr fbfbfbffjhfbfbfbfbfbfbf

Question 2
Find the volume.

Answers

Answer:

Volume of cone = 686π/3 or 718.67 in³

Step-by-step explanation:

Given the following data;

Radius, r = 7 in

Height, h = 14 inches

From the diagram, we can see that the object is a cone

To find the volume of a cone;

Mathematically, the volume of a cone is given by the formula;

[tex] V = \frac{1}{3} \pi r^{2}h[/tex]

Where;

V is the volume of the cone.

r is the radius of the base of the cone.

h is the height of the cone.

Substituting into the equation, we have;

[tex] Volume = \frac{1}{3} * \frac {22}{7} *7^{2}*14 [/tex]

[tex] Volume = \frac{1}{3} * 22 * 7 * 14 [/tex]

[tex] Volume = \frac{1}{3} * 2156 [/tex]

[tex] Volume = 718.67 [/tex]

Volume of cone = 718.67 in³ or 686π/3 in³

The London Eye Ferris wheel has a diameter of 120 meters. It completes one revolution in 30 minutes. About how far will a person travel along the arc of the Ferris wheel in 10 minutes while riding the London Eye?

Answers

Answer:

Step-by-step explanation:

PLSS HELP!! I NEED TO FIND THE SURFACE AREA OF THIS CYLINDER!!

Answers

Answer:

338π

Step-by-step explanation:

[tex]2\pi rh+2\pi r^2[/tex]

[tex]2\pi *7*20+2\pi *7^2=240\pi +98\pi[/tex]

Helppppppp pleaseeeeeeeee

Answers

Answer:

x = 8

Step-by-step explanation:

m<PQR = (2x + 6)°

m<RPQ = (x - 7)°

m<QRS = (5x - 17)°

<PQR and <RPQ are interior angles opposite to the exterior angle <RPQ.

Therefore,

m<PQR + m<RPQ = m<QRS (Exterior Angle Theorem of a Triangle)

Substitute in the values

2x + 6 + x - 7 = 5x - 17

Add like terms

3x - 1 = 5x - 17

Collect like terms

3x - 5x = 1 - 17

-2x = -16

-2x/-2 = -16/-2

x = 8

Anquan's account balance is $29.96, and Will's account balance is -$29.96. Which of the following is true

Answers

Answer:

is there anymore to the question because this doesn't make sense

Step-by-step explanation:

Can someone please answer and explain this.

Answers

Answer:

x =74

Step-by-step explanation:

540 degrees is the sum of all the angles

90 +90 + 2x+10 + x + 2x-20 = 540

Combine like terms

5x+170=540

Subtract 170 from each side

5x = 370

Divide by 5

5x/5 = 370/5

x =74

a cylinder water tank carries a volume of 150L when it is full. The water tank has a base surface with a diameter of 140 cm. what is the height of the water tank?​

Answers

Answer:

9.74418019cm=h

~ 9.7442

Step-by-step explanation:

volume of a cylinder=πr²h

radius=½of diameter

=½×140cm

=70cm

1l=1000cm³

150l=150l/1l×1000cm³

=150000cm³

150000cm³=π(70cm)²×h

150000cm³=15393.8040cm²h

150000cm³/15393.8040cm²=15393.8040cm²h/15393.8040cm²

150000cm³/15393.8040cm²=h

9.74418019cm=h

Evaluate the following expression. "12 more than 5"

Answers

Answer:

17

Step-by-step explanation:

12+5

Get brainiest if right!!
10 points

Answers

Answer:

the ordered list is x, x+2, x+4, x+6, x+8, x+10

formula is Tn = 2n - 2

Determine the value of x.

Answers

The answer to this question is B) 44.78

Answer:

Step-by-step explanation:

x = 44.78

Other Questions
The sum of 4 and y divided by 5 equal to 9. Find the value of y. Find the slope of the line that passes through these two points. (0, -2): (5. 3) Which line from the excerpt is an example of foreshadowing? Not Billy Mason Farrell, just Billy Mason. "What?" he tried to scream . . . The door across the landing was still open. A soft light filled the room . . . 3 disadvantages a of engaging in sexual activities outside of marriage answer Read the following graduated cylinder.How many ML? Plz help its due very soon Select the correct answer. Which of the following is not a component of physical fitness? A. The ability of your body to move through a full range of motion B. Your muscles' ability to continue performing without fatigue C. How much you weigh D. Your hearts and lungs' ability to supply oxygen to your body during physical activity Reset Next Which verb best completes the following sentence? Mis padres y yo ________ de Colorado. A. somos B. estamos C. son D. estoy how many 1/4 doses can be drawn from a 7 and 1/2 vial of medication HELP HELPP PLS QUICK What would be the weight (in Newtons) of a person with a mass of 80 kg on Earth, where the acceleration due to gravity is approximately 9.8 m/s/s? *please help asap What is the period of the graph of y5cos(4x + 1) + 3? Q= Which one of the following statement is incorrect?A. When a switch is closed in a circuit, no current flows.B. A switch is a device for turning on and off an electric current.C. An electric current is a complete loop around which a current can flow.D. An electric current is a flow of charge Vai tr ca yu t thi c vi s thng li ca cuc cch mng thng 8 Why do powders react faster than lumps?1)Particles of a powder have a lower activation energy2)Collisions between reactant particles are more frequent3)Particles of a powder move faster 3. Solve the system of equations without graphing. Show your reasoning. 2y = x - 44x + 3y = 5 A triangle has sides with lengths of 5 kilometers, 11 kilometers, and 12 kilometers. Is it a right triangle? pa answer posa music ko po yan 50 point kung Tama brainless narinpo :) Would cloning be considered asexual or sexual reproduction?ANSWERR ASPP PLS which statment describes an outcome of the hundred years war